LSAT and Law School Admissions Forum

Get expert LSAT preparation and law school admissions advice from PowerScore Test Preparation.

User avatar
 Dave Killoran
PowerScore Staff
  • PowerScore Staff
  • Posts: 5850
  • Joined: Mar 25, 2011
|
#87691
Complete Question Explanation
(The complete setup for this game can be found here: lsat/viewtopic.php?t=1469)

The correct answer choice is (B)

This question asks you to identify the pair of courses in which at least one of the two courses must always be taken. Courses that have more restrictions—such as M—are not attractive candidates to appear in the correct answer because they are difficult to pair with other variables. If the pair listed must always have at least one course in the group of three or four courses taken, then variables with fewer restrictions are more likely candidates than variables with more restrictions. Thus, using that analytical principle, L and T is the most likely pairing for the correct answer because L and T are the two least restricted variables in the game.

Answer choice (B) contains L and T, and ultimately (B) is the correct answer. But, if we did not use the analytical pattern above, is there another way to arrive at this answer? Yes. One method is to use the prior hypotheticals to eliminate answer choices. Using this method, we can use the solution to question #18 (H, P, T) to eliminate answer choice (C). The solution to question #20 (L, S, T) eliminates answer choice (D). And the solution to question #21 (M, L, S/W) eliminates answer choice (A). Thus, using hypotheticals would eliminate every answer choice except (B) and (C). You could then make a hypothetical for answer choice (B) or (C) to determine which was correct (to make a hypothetical that proves or disproves an answer, attempt to make a viable solution that uses neither of the two courses in the answer. If a viable solution can be found, the answer is incorrect; if a viable solution cannot be found, the answer is correct).
 jgabalski
  • Posts: 16
  • Joined: Feb 16, 2017
|
#34228
Can someone please provide me a full justification for why the answer to this question is B? I understand the setup and significance of M as the most powerful variable. I was really only to get to this answer by process of elimination from the other pairs where neither of the variables HAD to be selected. Thank you.
 Charlie Melman
PowerScore Staff
  • PowerScore Staff
  • Posts: 85
  • Joined: Feb 10, 2017
|
#34248
Hi J,

This question asks us for a pair of things that cannot be in the out group together; that is, a pair that the student cannot not take at the same time. If you put L and T out at the same time, then there aren't enough things that can go in—that the student can take. No matter what you try to put in the group of classes the student can take, the rules will always exclude one too many variables. You can prove this out by trying all the possibilities that you have when L and T are out.
 amagari
  • Posts: 23
  • Joined: May 09, 2017
|
#35462
I didn't understand the language in the setup of this question. I understood it to mean that what two items must be selected. But in the explanation, the wording was interpreted to mean what two items can never be unselected together. I understand now that I've heard the explanation.

But what dictated arriving at that conclusion? Why think about it like that instead of some other way?
User avatar
 Jonathan Evans
PowerScore Staff
  • PowerScore Staff
  • Posts: 726
  • Joined: Jun 09, 2016
|
#35601
Hi, Amagari,

Good question. This "must be true" question hinges on the logical "or" (which this problem actually makes explicit)—"must take one or the other or both of"—that is, a non-exclusive "or." The condition would be met with either one of the options by itself or both of the options together.

Ask yourself, what is the situation that would contradict what this question is asking for? What do we want not to happen? We don't want for it to be possible to exclude both choices from the selection, e.g. it is possible to have a selection with neither history nor statistics; therefore, that is an incorrect answer.

The correct answer will have two choices at least one of which must be chosen.

Thus, to answer your question, when faced with an AR question, ask yourself both what would satisfy the question (what do you want in a correct answer choice), and ask yourself what would fail to satisfy the question (what do you not want, what will happen in the incorrect answer choices).

For a strategy note, on this question (and on this game), it is important to observe that all the conditional clues indicate some form of double-not-arrow relationships. In other words, lots of these choices don't work with each other. Combine this fact with the requirement that we have at least three options in our selection. Thence you can conclude that the fact that there are so many restrictions means we need to pay especially close attention to our "free agents"; the choices without restrictions will be very important to fill the minimum required slots.

How does this help you here? If you are looking for a minimum possible selection possibility, it is highly likely that L (the free agent) will be involved.

I hope this helps!
User avatar
 parisielvirac
  • Posts: 30
  • Joined: Jan 20, 2021
|
#98070
answer choice C was elimnated in the answer above.

Do you mean use hypotheticals to see whether B or D is the correct answer?

(reffering to botom of answer, maybe there was a typo.)
 Luke Haqq
PowerScore Staff
  • PowerScore Staff
  • Posts: 722
  • Joined: Apr 26, 2012
|
#98486
Hi parisielvirac!

It seems like that was intended to narrow down the answers to (B) and (E). One can eliminate answer choices (A), (C), and (D) by looking to questions 21, 18, and 20.

Get the most out of your LSAT Prep Plus subscription.

Analyze and track your performance with our Testing and Analytics Package.